Based on the passage, it can be concluded that the author and Broyles-González hold essentially the same attitude toward

joseph on December 10, 2017

again conditionals

there are a lots of conditional indicators in the last example as there are quantifiers. why did we or why are we not able to diagram any conditional for the question?

Replies
Create a free account to read and take part in forum discussions.

Already have an account? log in

Mehran on February 10, 2018

Because of the quantifiers that also appear in these statements which takes them out of the S & N realm.

For example, "Only a few always do their own research before investing."

While this does have "only" it is not a S & N statement due to the presence of "few."

Hope that helps!

Robert on May 29, 2019

Hey, so I can get all the contrapostives from the drill practice I can get the premise contrapostives and the conclusion contrapostibes but I still don't know how to get the missing premise ?
example
d ->a
c->d
missing premise ? comes from where
Not x->a

Ravi on May 30, 2019

@sigmajonez14,

Great question. Let's take a look.

D - >A (not A - >not D)
C - >D (not D - >not C)
:
Conclusion: Not X - >A (not A - >X)

We need to add a premise that allows us to chain everything up.

not A - >not D - >not C

We need to go from not A - >X, so if we add not C - >X, we can arrive at
our conclusion.

The missing premise is not C - >X (not X - >C).

Does this make sense? Let us know if you have any questions!

Robert on May 30, 2019

on the missing premise do we just put in X
from the contrapostives could it be not a ->not d->not c->x

basically you are just working from the contrapostives and trying to link the first cp with the last and link them

Ravi on May 31, 2019

@sigmajonez14, right. We look at the original and the contrapositive for each premise and the conclusion and figure out how to best chain it all together.

on July 4, 2021

Just for clarification, we cannot reverse and negate the "some" quantifiers, correct? (example 1)

example: WMBC - some - PC

Cannot be written = not PC - some - not WMBC

Correct?

on December 8, 2021

Question 6 has both without and only statements, which should introduce a S&N statement. How do we know (other than the context of the lesson) that we should diagram these clauses as most statements rather than S&N statements?

Ravi on February 5, 2022

@JTempleton88, that's correct.

@Jolson0023, we know we should diagram these clauses as most statements because even though they have without and only statements, they also include the word "most." Any time you see a quantifier, you know the sentence you're looking at needs to be diagrammed as such.